Last visit was: 26 Apr 2024, 21:26 It is currently 26 Apr 2024, 21:26

Close
GMAT Club Daily Prep
Thank you for using the timer - this advanced tool can estimate your performance and suggest more practice questions. We have subscribed you to Daily Prep Questions via email.

Customized
for You

we will pick new questions that match your level based on your Timer History

Track
Your Progress

every week, we’ll send you an estimated GMAT score based on your performance

Practice
Pays

we will pick new questions that match your level based on your Timer History
Not interested in getting valuable practice questions and articles delivered to your email? No problem, unsubscribe here.
Close
Request Expert Reply
Confirm Cancel
SORT BY:
Date
Tags:
Show Tags
Hide Tags
Manager
Manager
Joined: 02 Nov 2018
Status:Manager
Posts: 213
Own Kudos [?]: 877 [4]
Given Kudos: 110
Location: Bangladesh
Send PM
RC & DI Moderator
Joined: 02 Aug 2009
Status:Math and DI Expert
Posts: 11181
Own Kudos [?]: 31969 [0]
Given Kudos: 291
Send PM
VP
VP
Joined: 28 Jul 2016
Posts: 1212
Own Kudos [?]: 1728 [0]
Given Kudos: 67
Location: India
Concentration: Finance, Human Resources
Schools: ISB '18 (D)
GPA: 3.97
WE:Project Management (Investment Banking)
Send PM
RC & DI Moderator
Joined: 02 Aug 2009
Status:Math and DI Expert
Posts: 11181
Own Kudos [?]: 31969 [0]
Given Kudos: 291
Send PM
Re: There are exactly N distinct rational numbers k such that |k| < 200 an [#permalink]
Expert Reply
globaldesi wrote:
chetan2u wrote:
Noshad wrote:
There are exactly N distinct rational numbers k such that \(|k| < 200\) and \(5x^2 + kx + 12 = 0\)
has at least one integer solution for x. What is N?

(A) 6

(B) 12

(C) 24

(D) 48

(E) 78



Let us take k to one side => \(5x^2 + kx + 12 = 0........k=-5x-\frac{12}{x}............\)
Now |k|<200 or -200<k<200, that is \(-200<-5x-\frac{12}{x}<200\)
-5x will become equal to 200, the moment x is 40, so x has to be less than 40..
Similarly -5x will become greater than 200, the moment x is -40, so x has to be greater than -40..

Thus x are from -39 to 39, which equals 2*39+1=79, which includes k as 0.
However k as 0 is not possible as the equation is not defined at that point =>\(5x^2 + kx + 12 = 0..........5x^2+12=0.....5x^2=-12\) NOT possible as \(5x^2\geq{0}\)
Thus 79-1=78

E


Chetan2u can you explain the highlighted part.
why does -5x becoming 400 is a problem. Since K will still b -5x-12/x which is less than 400
Similarly for next stmt


-5x will become 200 when x is -40, but -12/x will also become positive (-12)/(-40)=12/40, so sum is 200+(12/40) >200
Similarly for the other case..
-12/x will have SAME sign as -5x, so will get added, either both as negative, then <-200 or both as positive, then >200
User avatar
Non-Human User
Joined: 09 Sep 2013
Posts: 32692
Own Kudos [?]: 822 [0]
Given Kudos: 0
Send PM
Re: There are exactly N distinct rational numbers k such that |k| < 200 an [#permalink]
Hello from the GMAT Club BumpBot!

Thanks to another GMAT Club member, I have just discovered this valuable topic, yet it had no discussion for over a year. I am now bumping it up - doing my job. I think you may find it valuable (esp those replies with Kudos).

Want to see all other topics I dig out? Follow me (click follow button on profile). You will receive a summary of all topics I bump in your profile area as well as via email.
GMAT Club Bot
Re: There are exactly N distinct rational numbers k such that |k| < 200 an [#permalink]
Moderators:
Math Expert
92948 posts
Senior Moderator - Masters Forum
3137 posts

Powered by phpBB © phpBB Group | Emoji artwork provided by EmojiOne